A is plotted on a coordinate grid at start bracket 3 and 1 over 2, negative 1 end bracket.. Choose the correct grid that best shows the location of the point. A coordinate grid that shows a point located 4 units to the left and 1 unit down from the origin. A coordinate grid that shows a point located halfway between 3 and 4 units to the right and 1 unit down from the origin. A coordinate grid that shows a point located halfway between 3 and 4 units to the left and 1 unit up from the origin. A coordinate grid that shows a point located halfway between 3 and 4 units to the right and 1 unit up from the origin.

Answers

Answer 1

Answer:

im so sorry for the other answer i reported it and im gonna help with this question because i have the same question on my test.

ok so i got C it makes the most sense to me but if im wrong then ill delete this.

If im correct please mark me as brainliest

Step-by-step explanation:

yes this is wrong but i cant delete it so just ignore it

A Is Plotted On A Coordinate Grid At Start Bracket 3 And 1 Over 2, Negative 1 End Bracket.. Choose The
Answer 2

Answer:

B  3 1/2 to the right and one unit down

Step-by-step explanation:


Related Questions

Math Question (10 Points):

Answers

The correct answer is A

Answer:

Step-by-step explanation:

We need the two numbers to tell us where the point is on the coordinate plane. The first number tells us where it is on the x-axis, and the second number tells us where it is on the y-axis. The very middle of the coordinate plane is the point (0, 0)

Based on this the answer is horizontol plane and x coordinate

Patrick graphed (shown below) the high temperatures (in °C) that he experienced on each day of his trip to
Homer, Alaska
What is the meaning of point A?​

Answers

Answer:

(7, -5.5)

Step-by-step explanation:

is seen below on this graph (7, -5.5) shows x = 7 and y = -5.5

(7, 5.5) look ^^ they should help u

help me i need help i am bad at math

Answers

Answer:

4x10⁷is more than 4x10¹⁰

10^4
Because
The equation is the same with different powers
10^7 - 10^3

7-3=4

Please help ASAP!!!!!!!

Answers

Answer:

  $86.81

Step-by-step explanation:

Using the given formula, we want to compute A for ...

  P = 4750

  r = 0.2279

  n = 365 . . . . . assuming "exact" interest

  t = 1 or 30

For 1 day late:

  A = 4750(1 +0.2279/365)^(365·(1/365)) = 4752.97

For 30 days late:

  A = 4750(1 +0.2279/365)^(365·(30/365)) = 4839.78

The difference in these payment amounts is ...

  $4839.78 -4752.97 = $86.81

You would save $86.81 in interest charges by paying only 1 day late.

_____

Comment on the question

It would be a poor choice of credit card to use one that compounds interest daily. Most do so on a monthly basis.

Answer 86.61 !


Explanation : copied other person if you don’t mind

What is 3 divided by 2,127

Answers

●✴︎✴︎✴︎✴︎✴︎✴︎✴︎✴︎❀✴︎✴︎✴︎✴︎✴︎✴︎✴︎✴︎✴︎●

       Hi my lil bunny!

❧⎯⎯⎯⎯⎯⎯⎯⎯⎯⎯⎯⎯⎯⎯⎯⎯⎯⎯⎯⎯⎯⎯⎯⎯⎯⎯⎯⎯⎯⎯⎯⎯⎯⎯⎯⎯⎯⎯☙

[tex]3\ \div \ 2127[/tex]

[tex]= \frac{1}{709}[/tex] (Decimal: 0.00141)

❧⎯⎯⎯⎯⎯⎯⎯⎯⎯⎯⎯⎯⎯⎯⎯⎯⎯⎯⎯⎯⎯⎯⎯⎯⎯⎯⎯⎯⎯⎯⎯⎯⎯⎯⎯⎯⎯⎯☙

●✴︎✴︎✴︎✴︎✴︎✴︎✴︎✴︎❀✴︎✴︎✴︎✴︎✴︎✴︎✴︎✴︎✴︎●

If this helped you, could you maybe give brainliest..?

❀*May*❀

I think the answer is 709

Describe the transformations that will map triangle A to triangle B and illustrate the similarity between the two triangles.


A) reflect triangle a across the y-axis and then translate triangle a 5 units down.  B) reflect triangle A across the x-axis and then reflect triangle a across the y-axis.  
C) translate triangle a 4 units down and then rotate triangle A 180° counterclockwise about the origin. 
D) reflect triangle a across the x-axis and then rotate triangle A 90° counterclockwise about the origin

Answers

Honestly I’m not for sure but I’m think it’s either a or b

The triangle is shifted 4 units downwards and then is rotated about the origin.

What is image rotation?A rotation is a type of transformation which is a turn.A figure can be turned clockwise or counterclockwise on the coordinate plane.In both transformations the size 3 shape of the figure stays exactly the same.

Given are triangles A and B.

Transformation 1 : The triangle is shifted 4 units downwards.

Transformation 2 : The figure is rotated about the origin.

Therefore, the triangle is shifted 4 units downwards and then is rotated about the origin.

To solve more questions on dilation, visit the link below -

brainly.com/question/2859427

#SPJ5

Someone pls help . Thank you sm ! i forgot how to do this lol .

Answers

Answer:

x = - 20, h = 34

Step-by-step explanation:

i can't tell ypou what the class period is, but for -x = 20, multiply each side by -1, then you get x = -20

-32 = 2-h

+32 .  +32

0=34-h

+h .   +h

34=h

flip

h=34

X= -20 and H= 34 the first one you just carry the negative sign to the 20

PLEASE HELP ME WITH THIS QUESTION FOR HOMEWORK!!!!!!!!!!

Answers

Answer:

Angle MON is equal to 130 degrees.

Step-by-step explanation:

We are told LON is 180 degrees.

Angles LOM and MON are supplementary meaning they add to 180 degrees.

Using this information we know 12x+120=180.

We can use the subtractive property of equality to subtract 120 from both sides.

Do thins we get 12x=60.

Using the division property of equality we can divide both sides by 12 to get x=5.

We can then use the substitution property of equality to get MNO=8(5)+90.

We can then simplify to get MNO=130.

Angle min is 130 degrees

m∠1=36°, m∠4=49°, m∠6=131°. Find m∠9. A. 144 B. 242 C. 216 D. 36

Answers

Answer:

<9 =144

Step-by-step explanation:

<1 + <4 + < 6 + <9 = 360

36+ 49+ 131 + <9 = 360

216 +  < 9 = 360

< 9 = 360-216

<9 =144

Answer:

A. 144

Step-by-step explanation:

Since m<6 + m<4 = 131 + 49 = 180, then FE is a line.

Angles 1 and 9 are a linear pair, so they are supplementary.

m<9 + m<1 = 180

m<9 + 36 = 180

m<9 = 144

Town B is 250 km from town A on a bearing of 080°.
Town C is 250 km from town B on a bearing of 220°.
What is the bearing from town A to town C?

Answers

Answer:

Step-by-step explanation:

220-80
The answer would be 140

7/13 + 1/5 (its in fractions)

Answers

Answer:

48 ÷ 65

Step-by-step explanation:

hope it helps you

Answer:

[tex]\frac{48}{65}[/tex]

Step-by-step explanation:

Step 1: Find LCM for denominators

13(5) = 65

So we need to convert our fractions to add them

Step 2: Convert [tex]\frac{7}{13}[/tex] by multiplying both top-bottom by 5

[tex]\frac{7}{13} (\frac{5}{5} ) = \frac{35}{65}[/tex]

Step 3: Convert [tex]\frac{1}{5}[/tex] by multiplying both top-bottom by 13

[tex]\frac{1}{5} (\frac{13}{13} ) = \frac{13}{65}[/tex]

Step 4: Add the converted fractions

[tex]\frac{13}{65} +\frac{35}{65} =\frac{48}{65}[/tex]

And we have our answer!

For lunch, you get tacos that cost $5.50. You also get chips and salsa for $3.99. What is the total cost of your lunch?

Answers

Answer:

9.49

Step-by-step explanation:

Add the price of the chips and salsa to the price of the tacos

5.50

+3.99

---------------

9.49

Answer: $9.49

Step-by-step explanation:

0+9 is 9 and 5+9 is 14 but carry the 1 to the left and add 5+3+1 and you get your answer $9.49

$5.50

+$3.99

————

$9.49

6.21(3.1) answer for number 6

Answers

The answer is 19.251

The polynomial function f (x) = 5 x Superscript 5 Baseline + sixteen-fifths x minus 3 is graphed below. On a coordinate plane, point P is shown on the graph of a function. Point P is at (0.6, 0). Which is a potential rational root of f(x) at point P?

Answers

Answer:

3/5

Step-by-step explanation:

What divisor is represented by the synthetic division below? x + 5. One factor of  x^3+x^2+x+1. If f(-5) = 0, what are all the factors of the function mc022-... -4 and 3. The polynomial function f(x) = 5x5 + 3x - 3 is graphed below. The root at point P maybe 3/5  3.

The potential rational roots are: [tex]\pm 1, \pm \frac{1}{5}, \pm 3,\pm \frac{3}{5}[/tex]

What is the rational root theorem?

The rational root theorem is used to determine the possible rational roots of a polynomial function

The equation of the function is given as:

[tex]f(x) = 5x^5 +\frac{16}{5} x -3[/tex]

Considering a polynomial function, f(x).

Such that:

[tex]f(x) = px^n + .......... +q[/tex]

The possible rational roots are:

[tex]Roots =\pm \frac{Factors\ of\ q}{Factors\ of\ p}[/tex]

So, we have:

[tex]Roots =\pm \frac{Factors\ of\ 3}{Factors\ of\ 5}[/tex]

List the factors of 3 and 5

[tex]Roots =\pm \frac{1,3}{1,5}[/tex]

Split

[tex]Roots =\pm \frac{1}{1}, \pm \frac{1}{5}, \pm \frac{3}{1},\pm \frac{3}{5}[/tex]

Simplify

[tex]Roots =\pm 1, \pm \frac{1}{5}, \pm 3,\pm \frac{3}{5}[/tex]

Hence, the potential rational roots are:

[tex]\pm 1, \pm \frac{1}{5}, \pm 3,\pm \frac{3}{5}[/tex]

Read more about potential rational roots at:

https://brainly.com/question/7594092

Describe the end behavior of the graph of f(x) = x^3 (x + 3)(-5x + 1) using limits.

Answers

Answer: Rises to the left and falls to the right

Step-by-step explanation:

Use the degree and the leading coefficient to determine the behavior.

There is a good video called "Describing End Behavior Using Limit Notation" by Mario's Math Tutoring, and it helps to show you how to find the end behavior with limits.

rises to the left, falls to the right

if
[tex] log_{x} \: a \: + log_{x} \: \frac{1}{a} = t[/tex]
what is the possible value of t?​

Answers

Answer:

t= 0

Step-by-step explanation:

Using the rules of logarithms

log x + log y = log(xy)

[tex]log_{b}[/tex] 1 = 0

Given

[tex]log_{x}[/tex] a + [tex]log_{x}[/tex] [tex]\frac{1}{a}[/tex] = t ,then

[tex]log_{x}[/tex] (a × [tex]\frac{1}{a}[/tex] ) = t

[tex]log_{x}[/tex] 1 = t , thus

t = 0

I’m pretty sure the answer is t=0

you move up 7 units and left 6 units. you end at (-1,5). where did you start?​

Answers

Answer:I beleive it is (5,-2)

Step-by-step explanation:

First you wanna go backwards so start from (-1,5) and go down 7 units and 6 units right then you’ll get where you started ❤️ hope this helped

I NEED THE ANSWER ASAP

The number of students who smoke cigarettes at Broxton College is decreasing at a rate of one smoker every 6.31 days. At what rate in smokers per year is the number of smokers declining? Assume 365 days in a year and round to the nearest tenth of a smoker per year

Answers

Answer:

57.8

Step-by-step explanation:

We can set up a proportion for this, assuming x is the smokers lost in 365 days.

[tex]\frac{1}{6.31} = \frac{x}{365}[/tex]

Using the cross products property, we know that x will be equal to:

[tex](365\cdot1) \div 6.31\\365\div6.31\\\\\approx 57.8[/tex]

Hope this helped!

57.8 is the answer to this problem

Find the value of y and x.

Answers

Answer:

x = 130°y = 65°

Step-by-step explanation:

All inscribed angles intercepting the same arc have the same measure.

If you slide the vertex of angle y clockwise around the circle until it coincides with the vertex of the angle marked 65°, you see that those two inscribed angles (y and 65°) intercept the same arc, so have the same measure:

  y = 65°.

The measure of an inscribed angle is half the measure of the intercepted arc, so the arc marked x is double the angle 65°.

  x = 130°.

The answer for this is y= 65 degrees and x= 130 degrees

Sally’s house is located at (5, −1) and her school is located at (−9, 7). Her best friend Molly lives at
the midpoint of Sally’s house and school. What coordinate represents Molly’s house?
A. (2, −3) B. (−2, 3) C. (3, −2) D. (−3, 2)

Answers

Answer:

B. (−2, 3)

Step-by-step explanation:

To find the midpoint of a segment, add the x-coordinates of the endpoints and divide by 2. Add the y-coordinates of the segments and divide by 2.

x: (5 + (-9))/2 = -2

y: (-1 + 7)/2 = 3

Midpoint: (-2, 3)

Answer: B. (−2, 3)

Answer:

b

Step-by-step explanation:

-9+5=-4÷2=-2

-1+7=6÷2=3

2. Solvex2 - 6x = -5 by completing the square , Show all work for the steps below . ( a ) Forx ? - 6x + = -5+ , what value of c is used to complete the square ? ( b ) Substitute the value for c in Part 2 ( a ) . Then complete the square to rewrite the equation as the square of a binomial . ( c ) Solve for x

Answers

Answer:

see explanation

Step-by-step explanation:

Given

x² - 6x = - 5

To complete the square

add ( half the coefficient of the x- term )² to both sides

x² + 2(- 3)x + 9 = - 5 + 9 [ part a, c = 9 ]

(x - 3)² = 4 [ part b ]

Take the square root of both sides

x - 3 = ± [tex]\sqrt{4}[/tex] = ± 2 ( add 3 to both sides )

x = 3 ± 2

Thus part c is

x = 3 - 2 = 1

x = 3 + 2 = 5

What is the area of the polygon shown below? (in the image). A. 322 mm^2 B. 364 mm^2 C. 520 mm^2 D. 584 mm^2 Show all work please!

Answers

Answer:

A.

Step-by-step explanation:

Find the area of the triangle: 26-20=6. 14x6=84/2=42. Now that we calculated the triangle's area, we need to find out the rectangle's. 20x14=280. Add them up: 280+42=322 mm^2. Triangle: b x h/2. Rectangle: l x h.

The area of a 2D form is the amount of space within its perimeter. The area of the given figure is 322 mm². The correct option is A.

What is an area?

The area of a 2D form is the amount of space within its perimeter. It is measured in square units such as cm², m², and so on. To find the area of a square formula or another quadrilateral, multiply its length by its width.

The given figure can be broken into a rectangle and a right-angle triangle, in order to calculate the area of the given polygon.

The area of the rectangle with a length of 20mm and height of 14 mm is,

Area of the rectangle = 20mm × 14mm

                                    = 280 mm²

The area of the right-angled triangle with a base length of 6mm and height of 14 mm is,

Area of the triangle = (1/2) × 6mm × 14mm

                                 = 42 mm²

Now, the total area of the given figure can be written as,

The total area of the given figure

= Area of the rectangle + Area of the triangle

= 280 mm² + 42 mm²

= 322 mm²

Hence, the area of the given figure is 322 mm².

Learn more about the Area here:

https://brainly.com/question/1631786

#SPJ5

Help, please show work

Answers

Answer:

132

Step-by-step explanation:

If US bisects the angle then

BUS = SUL

2x+10 = 3x-18

Subtract 2x from each side

2x+10 -2x = 3x-18-2x

10 = x-18

Add 18 to each side

10+18 =x

28 =x

We want to find BUL

BUL = BUS + SUL

      2x+10 + 3x-18

     5x-8

    5*28 -8

   140-8

   132

Answer:

m<BUL=132

Step-by-step explanation:

Since US bisects <BUL, we know that <BUS ≅ <SUL

This is based on the angle bisector definition.

Hence, we can set up an equation to solve for x.

<BUS≅<SUL

2x+10=3x-18

Subtract 10 from both sides

2x+10-10=3x-18-10

2x=3x-28

Subtract 3x from both sides

2x-3x=3x-3x-28

-x=-28

Divide both sides by -1

x=28

m<BUL is the sum of m<BUS and m<SUL

m<BUL=m<BUS+m<SUL

m<BUS=2x+10

m<SUL=3x-18

Plug it in

m<BUL=2x+10+3x-18

Combine like terms

m<BUL=5x-8

Plug in 28 for x

m<BUL=5(28)-8

m<BUL=140-8

m<BUL=132

During a certain 25-year period, the consumer price index (CPI) increased by 99%,but during the
next 25-year period, it increased by only 1%. Which of these conditions must have existed during the
second 25-year period?
A. Conflation
B. Deflation
C. Inflation
D. Stagnation​

Answers

I think the answer to this is D or stagnation

For the next 25-year period,(CPI) is increased by only 1% due to Inflation and Stagnation.

What is Stagnation ?

Stagnation is a situation of slow economic growth and relatively high unemployment (which basically means that aggregate production is reducing and some of the inputs of the economy, such capital or labor, are unemployed) , accompanied by rising prices, or inflation.

In terms of national accounting, it means a reduction in gross domestic product (GDP), with inflation (rise in all prices in the economy).

In terms of aggregate demand and supply models, stagnation is the result of a contraction of aggregate supply, which ceteris paribus, results in lower levels of production and higher prices.

Many theories have tried to explain this phenomena. Common interpretations link stagnation with and increase of the cost of production in the economy (that might be generated by an increase of gasoline for example), and its implications in the production (lower production because of higher costs), consequently unemployment and a rise of prices due to the increase of cost.

Here, During a certain 25-year period, the consumer price index (CPI) increased by 99% and during the next 25-year period, it increased by only 1% which shows slow economic growth which can be due to Inflation and Stagnation.

​Learn more about "Inflation" here:

https://brainly.com/question/28190771

#SPJ2

PLEASE HELP WILL GIVE BRAINLY!!!!!!!!!What is the vertex of the graph of the function f(x) = x2 + 8x − 2 ?
(−4, 18)
(0, -2)
(-8, -2)
(−4, −18)

Answers

Answer: D.  (-4, -18)

===================================================

Explanation:

A graph is a nice addition (and it will likely help us see the vertex directly), but it isn't necessary because we can use the equation given to us. Though I recommend using a graphing calculator to confirm the answer.

The original function is the same as y = 1x^2+8x + (-2). We see that it is in the form y = ax^2+bx+c where

a = 1

b = 8

c = -2

Use the values of 'a' and b to get the value of h, which is the x coordinate of the vertex.

h = -b/(2a)

h = -8/(2*1)

h = -4

The x coordinate of the vertex is x = -4. Plug this into the original equation to get

f(x) = x^2+8x-2

f(-4) = (-4)^2 + 8(-4) - 2

f(-4) = -18

Plugging x = -4 into f(x) leads to y = -18. The point (-4, -18) is on the parabola. Furthermore, this is the vertex (h,k)

------------

Alternatively, you can complete the square as shown below

y = x^2 + 8x - 2

y = (x^2 + 8x) - 2

y = (x^2 + 8x + 0) - 2

y = (x^2 + 8x + 16 - 16) - 2

y = (x^2 + 8x + 16) - 16 - 2

y = (x+4)^2 - 18

y = 1(x+4)^2 - 18

y = 1(x-(-4))^2 - 18

The last equation is in the form y = a(x-h)^2 + k with (h,k) = (-4,-18) being the vertex. The 16 is the result of taking half of 8 and squaring that result. We have 16-16 = 0 to make sure that we don't change the equation and keep things balanced. This is the same as adding 16 to both sides. All of this is done so we can end up with the (x+4)^2 perfect square portion.You can expand out (x+4)^2 - 18 and you should get x^2+8x-2 again.

D is the answer

Because you need to rewrite in vertex form and use this form to find the vertex (h,k)

Which should get you to

(-4,-18)

A vending machine accepted any combination of nickels, dimes, and quarters that added to $0.40. How many different combinations of coins were possible?

Answers

Answer: 1 quarter, 1 dime and 1 nickel.

Step-by-step explanation:

Quarter: 25 cents.

Dime: 10 cents

Nickel: 5 cents

25 + 10 = 35

35 + 5 = 40

Answer: 25 (1 quarter), 10 (1 dime), 5 (1 nickel) is 0.40 cents when all three numbers are summed up.

25 for one quarter and the other ones 5 cents and 10 cents

Please help with 2 and 3 for maths

Answers

Answer:2-4=

Step-by-step explanation:y-16-(-42)

y-16+16

y-32

y-16-(-22)

y-16+4

y-20

y-16-(02)

y-16

y-16-(42)

y-0

Answer:

The answer is 2-4

Step-by-step explanation:

find the value of a answers: a:15 b:14 c:19 d:16

Answers

Answer:

6a+10= 3a+55

6a-3a= 55-10

3a= 45

a= 45/3

a= 15

The answer is (A) 15

Hope this helps^°^

Answer:

A. 15

Step-by-step explanation:

The angles are vertical angles, therefore they are congruent. We can set the two measures equal to each other.

[tex]6a+10=3a+55[/tex]

Now, solve for a. We must get a by itself on one side of the equation.

First, subtract 3a from both sides of the equation.

[tex]6a-3a+10=3a-3a+55[/tex]

[tex](6a-3a)+10=55[/tex]

[tex]3a+10=55[/tex]

Next, subtraction 10 to both sides of the equation. We subtract because 10 is being added to 3a. The inverse of addition is subtraction.

[tex]3a+10-10=55-10[/tex]

[tex]3a=55-10[/tex]

[tex]3a=45[/tex]

3 and a are being multiplied. The inverse of multiplication is division. Divide both sides of the equation by 3

[tex]3a/3=45/3[/tex]

[tex]a=45/3[/tex]

[tex]a=15[/tex]

The value of a is 15 and choice A is correct.

P. The inventory of a store 2 years ago was worth $45,000. This year it is
worth $49,500. What is the percent increase? Show your work

Answers

Answer:10% increase

Step-by-step explanation:

49500-45000=4500

4500/45000=0.1

0.1x100=10

10% increase

Answer:

10%

Step-by-step explanation:

follow the rules of calculating percentages.

fractions, decimals, etc.

pls mark brainliest

Use the model below to find the area of the shaded region.

Answers

Answer:4/12

Step1- 3 times 4 is 12

Step2- count how many twelvths are shaded

Step3- 4 twelvths are shaded

Answer: 4/12

Answer:

C. 1/3 square yard

Step-by-step explanation:

Given:

The length of shaded area = 2/3 yardThe width of shaded area = 2/4 yard

To find:

The area of shaded region

Solution:

The shaded region is a rectangle. Area of a rectangle is the product of the length and width:

Area = 2/3 yard * 2/4 yard = 1/3 square yard

Answer choice: C. 1/3 square yard

Other Questions
Which notation is better to use? (Choose between 4,000,000,000,000,000 m and 4.0 1015 m) Manuel is saving money for college. He already has $250. He plans tosave another $50 per month,Regardless of how many months he saves, how much does he save eachmonth? what is the importance of personal computers in connecting to the internet ?^please I need an answer, PLEASE ^ The perimeter of a scalene triangle is 14.5 cm. The longest side is twice that of the shortest side. Which equation can be used to find the side lengths if the longest side measures 6.2 cm? Explain who the Spanish conquistadores are/were (quick summary would be extremely helpful!) Central to agency theory is the concern with problems that can arise between the principals who are the owners of the firm and the agents who are the people who are paid by outside consultants to perform a job on their behalf.a. Trueb. False Which question did the author of "How to Grow a School Garden" considerabout the audience before writing the project plan?OA. What locations are best for growing a school garden?OB. What type of climate is my audience most likely located in?O C. Which types of plants will my audience be most likely to grow?O D. What misconceptions might my audience have about gardening? What was the goal of the Crusades?O A. To regain the glory of RomeO B. To push the Moors out of SpainO C. To eliminate the Jews from JerusalemwO D. To push the Muslims out of Holy Lands Paula drives 130 miles in 2.5 hours. How far would she drive in 4.5 at the same speed?*Please answerI will award the Brainliest answer Which of the following is NOT characteristic of the Gothic style? Each lap around a park is 1 15 miles. Kellyn plans to jog at least 7 12 miles at the park without doing partial laps. How many laps must Kellyn jog to meet her goal? The focus of a parabola is (3,-7) and the directrix is y = -4.What is an equation of the parabola? Which option is NOT in the Link to portion of the Insert Hyperlink dialog box?Existing file or web pagePlace in this documentCreate a new documentPlace in another document A train moves at a speed of 90 km/hr. How far will it travel in 36 minutes? What doest this phrase from suggest? In the dying sunlight the people dragged, rather than their empty cotton sacks help me please ;))) (yes im very rich that's why I'm giving out lots of points -_- (and brainly ;)) which of the following segments have an error .Responsibilities to employers and family members are more important than schoolwork in many student's lives.employersstudent'slives a) employers b) student'sc) livesd)no error The braking distance, D, of a car is directly proportional to the square of its speed, v. When d=5, v=10Find d when v=70 A disk between vertebrae in the spine is subjected to a shearing force of 375 N. Find its shear deformation, taking it to have a shear modulus of 1.60109 N/m2. The disk is equivalent to a solid cylinder 0.750 cm high and 6.50 cm in diameter. Read and choose the correct verbs in the past tense to complete the sentence. Mientras el amo de llaves ________ el cuarto, la seora ________ su maleta. limpiaba; sac limpiaba; saca limpio; saca limpio; sac